Download as pdf or txt
Download as pdf or txt
You are on page 1of 24

CIVIL SERVICES (PRELIMINARY) TEST SERIES – 2019

TEST BOOKLET SERIES


ANSWER & EXPLANATION
Test Code:
P1-1921 GENERAL STUDIES
(English Version) TEST – 21

ANSWERS
Q. No. Answers Q. No. Answers Q. No. Answers Q. No. Answers
1 (c) 26 (d) 51 (b) 76 (c)

2 (c) 27 (c) 52 (b) 77 (c)


3 (b) 28 (c) 53 (b) 78 (b)
4 (d) 29 (a) 54 (d) 79 (c)
5 (b) 30 (a) 55 (a) 80 (b)
6 (d) 31 (c) 56 (a) 81 (a)
7 (a) 32 (c) 57 (c) 82 (c)
8 (a) 33 (d) 58 (a) 83 (c)
9 (c) 34 (c) 59 (c) 84 (b)
10 (d) 35 (a) 60 (c) 85 (c)
11 (c) 36 (a) 61 (b) 86 (d)
12 (a) 37 (c) 62 (d) 87 (c)
13 (c) 38 (b) 63 (c) 88 (b)

14 (d) 39 (a) 64 (c) 89 (c)


15 (b) 40 (d) 65 (a) 90 (b)
16 (b) 41 (b) 66 (b) 91 (b)
17 (c) 42 (b) 67 (a) 92 (c)
18 (d) 43 (a) 68 (d) 93 (b)
19 (a) 44 (a) 69 (d) 94 (b)
20 (c) 45 (a) 70 (a) 95 (c)
21 (a) 46 (d) 71 (b) 96 (c)
22 (b) 47 (a) 72 (a) 97 (d)
23 (a) 48 (d) 73 (b) 98 (d)
24 (b) 49 (c) 74 (b) 99 (b)
25 (a) 50 (b) 75 (d) 100 (a)
1. (c) zzassets consisting of cash and cash equivalents; or
Exp.: The Rig-Veda includes more than a thousand hymns, zzassets consisting of any amount of cash and cash
called sukta. These hymns are in praise of various gods equivalents and nominal other assets.
and goddesses. Hence, statement 1 is correct. Theoretically, shell companies are companies without
zzThree gods are especially important: Agni, the god of fire; active business operations or significant assets. Hence,
Indra, a warrior god; and Soma, a plant from which a
statement 2 is correct.
special drink was prepared.
Value addition: in August 2017, the SEBI directed stock
zzThese hymns were composed by sages (rishis). Priests
taught students to recite and memorise each syllable, exchanges to initiate action against 331 suspected shell
word, and sentence, bit by bit, with great care. companies and bar them from trading. Further, MCA
zzMost of the hymns were composed, taught and learnt by cancelled the registration of around 2,09,032 defaulting
men but a few of them were composed by women also. companies and the Ministry of Finance directed banks to
Hence, statement 2 is correct. restrict operations of bank accounts of such companies
2. (a) by the directors of such companies or their authorized
zzAngel tax was introduced in 2012 to arrest laundering of
representatives.
funds. Hence, statement 1 is correct. Recently, the Task Force on Shell Companies has
zzIt is a levy of 30.9 per cent on the amount exceeding the submitted its report to government.
fair market value of shares issued by unlisted companies 4. (d)
(mainly start-ups) which will be treated as income from
other sources. Hence, statement 2 is NOT correct: Exp.: The growth rates of agriculture & allied sectors have
zzThe government issued a notification in April 2018 to been fluctuating at 1.5% in2012-13, 5.6% in 2013-14, (-)
give exemption to startups under Section 56 of the Income 0.2% in2014-15, 0.7% in 2015-16 and 4.9% in 2016-17.
Tax Act in cases where the total investment including This uncertainties in grow thin agriculture are explained
funding from angel investors did not exceed `10 crore. by the fact that more than 50 percent of agriculture in
For the exemption, startups were also required to get
India is rainfall dependent which aggravate the production
approval from an inter-ministerial board and a certificate
of valuation by a merchant banker. risks. Hence, statement 1 is NOT correct.
zzFurther the exemption would apply only when the angel Item 2012-13 2013-14 2014-15 2015-16 2016-
investor had a minimum net worth of `2 crore or an 17 PE
average returned income of over `25 lakh in the preceding Growth
three financial years. in GVA
in
zzNews: Recently, many startup shave received notices to
Agricul-
pay angel tax since last year. ture &
3. (b) Allied
1.5 5.6 – 0.2 0.7 4.9
sec-
Exp.: In India shell companies are not defined under Companies tors at
Act 2013 or any other legislation. However, some laws 2011-12
can help curbing illegal activities such as money prices
laundering and can be used to target shell companies – (in per-
Benami Transaction (Prohibition) Amendment Act 2016, cent)
etc. Hence, statement 1 is NOT correct. The agriculture sector has been witnessing a gradual
Although, US has defined the shell companies under their
structural change in recent years. The share of livestock
Securities Act (Securities Act Rule 405 and Exchange
Act Rule 12b-2) as a Shell Company as a company, other in GVA in agriculture has been rising gradually, the share
than an asset-backed issuer, with no or nominal of the crop sector in GVA has been on the decline from
operations; and either: 65per cent in 2011-12 to 60 per cent in 2015-16. Hence,
zzno or nominal assets statement 2 is NOT correct.

641, First Floor, Dr. Mukherjee Nagar, Delhi-110009 2


Contacts% 011-47532596, (+91) 8448485517
E-mail: drishtiacademy@gmail.com, Web: www.drishtiias.com Facebook: facebook.com/drishtithevisionfoundation
Copyright – Drishti The Vision Foundation
https://telegram.me/pdf4exams https://telegram.me/ias201819

5. (b) 8. (a)
Exp.: Minimum Support Price (MSP) is a form of market Exp.: Since 1992, UNESCO has recognised a ‘mixed’ category
intervention by the Government of India to insure that tries to address the limitations of trying to divide sites
agricultural producers against any sharp fall in farm into ‘cultural heritage’ and ‘natural heritage’. This mixed
prices. category is one that protects cultural landscapes or areas
The minimum support prices are announced by the that show ‘significant interactions between humans and
Government of India at the beginning of the sowing the natural environment’.
season for certain crops on the basis of the recommendations
zzThe Khangchendzonga National Park (KNP) was
of the Commission for Agricultural Costs and Prices
(CACP). Hence, statement 1 is NOT correct. inscribed as India’s first ‘Mixed World Heritage Site’ in
CACP considers several factors while recommending the the UNESCO list on July 17, 2016. Currently, it is the
MSP for a commodity such as cost of production, changes only site in India to be identified as Mixed World Heritage
in input prices, international price situation, inter-crop Site. Eighty six per cent of the core lies in the Alpine zone
price parity, etc. Government announces minimum and the remaining portions are located in the Himalayan
support prices (MSPs) for 22 mandated crops and fair wet temperate and subtropical moist deciduous forest.
and remunerative price (FRP) for sugarcane. Hence, option (a) is correct.

The government procurement at MSP is highly skewed 9. (c)
in favour of some northern States whereas highly
discriminated against eastern States where it is minimal Exp.: Kutiyattam is a Sanskrit theatre, which is practised in
or non-existent. the province of Kerala. It is one of India’s oldest living
MSP has put excessive focus on some crops such as wheat, theatrical traditions. Hence, statement 1 is correct.
paddy and sugarcane at the expense of pulses, oilseeds, zzIt originated more than 2,000 years ago, Kutiyattam
etc. Hence, statement 2 is correct. represents a synthesis of Sanskrit classicism and reflects
6. (d) the local traditions of Kerala.
Exp.: A Kolam is a geometric line drawing composed of curved zzIn its stylized and codified theatrical language, neta
loops. It is drawn around a grid pattern of dots. This abhinaya (eye expression) and hasta abhinaya (the
drawing is made by using rice flour, chalk, chalk powder language of gestures) are prominent.
or rock powder. It is widely practised by female family zzIt has been inscribed in the list of Intangible Cultural
members in front of their houses in south India, especially Heritage of Humanity of UNESCO in the year 2008.
in Tamil Nadu state. Hence, option (d) is correct. Hence, statement 2 is correct.
7. (a) 10. (d)
Exp.: Mathura School of arts flourished on the banks of river Exp.: Alvars and Nayanars were saints from Tamil Nadu. They
Yamuna. A large number of images dating back to the led some of the earliest bhakti movements mainly in the
Kushana Period is from Mathura.
sixth century. Alvars were those who ‘immersed’ in
zzA distinct way of sculpting practised at Mathura makes devotion to Vishnu and Nayanars those who were
the images found here different from those at other centres devotees of Shiva. Hence, statement 1 is correct.
in the country.
zzBoth Alvars and Nayanars saints travelled from place to
zzThe image of the Buddha from the Katra mound belongs
place singing hymns in Tamil in praise of their gods.
to the second century CE. It represents the Buddha with
Hence, statement 2 is correct.
two Bodhisattvas attendants. Also, it is the birth place of
Lord Krishna and has remained an important pilgrimage zzDuring their travels the Alvars and Nayanars identified
for Hindus. certain shrines as abodes of their chosen deities. Very
zzNot only Hinduism and Buddhism but Jainism also often large temples were later built at these sacred places.
flourished at Mathura. Jain community was very These developed as centres of pilgrimage. Singing
prosperous and all the 24 Tirthankaras were worshipped compositions of these poet-saints became part of temple
at Mathura. rituals in these shrines, as did worship of the saints’
Hence, option (a) is correct. images.

641, First Floor, Dr. Mukherjee Nagar, Delhi-110009 3


Contacts% 011-47532596, (+91) 8448485517
E-mail: helpline@groupdrishti.com, Web: www.drishtiias.com Facebook: facebook.com/drishtithevisionfoundation
Copyright – Drishti The Vision Foundation
https://telegram.me/pdf4exams https://telegram.me/ias201819

zzOne of the most striking features of these two bhakti 15. (b)
traditions was the presence of women devotees. The
Exp.: The Reserve Bank of India (RBI) has released a new
compositions of Andal, a woman Alvar, were widely sung.
lavender `100 currency note. The banknote highlights
Andal saw herself as the beloved of Vishnu. Another
the rich and diverse cultural heritage of India as it
woman named Karaikkal Ammaiyar, a devotee of Shiva,
adopted the path of extreme asceticism in order to attain prominently displays a photograph of ‘Rani-ki-vav’, a
her goal. Her compositions were preserved within the Step well built as an 11th century architectural wonder.
Nayanar tradition. Hence, statement 3 is correct. Hence, option (b) is correct.

11. (c) zzIt is located on the banks of the Saraswati River in


Gujarat’s Patan. This stepwell was built by the Solanki
Exp.: A Brahman named Basavanna in twelfth century led a
dynasty’s queen Udayamati in the 11th century as a
new movement in Karnataka. His followers were known
memorial to her deceased husband Bhimdev I.
as Virashaivas (heroes of Shiva) or Lingayats (wearers
of the linga). Hence, statement 1 is correct. zzIt is maintained by the Archaeological Survey of India

zzLingayats continue to be an important community in the (ASI), Rani-ki-vav was added to the UNESCO World
region to date. They worship Shiva in his manifestation Heritage Site list in 2014.
as a linga, and men usually wear a small linga in a silver 16. (b)
case on a loop strung over the left shoulder.
Exp.: Spinnaker
zzLingayats believe that on death the devotee will be united
It is the world’s largest supercomputer designed to work
with Shiva and will not return to this world. Therefore
in the same way as the human brain.The newly formed
they do not practise funerary rites such as cremation,
prescribed in the Dharmashastras. Hence, statement 2 is million-processor-core Spiking Neural Network
correct. Architecture (SpiNNaker) machine is capable of
completing more than 200 million million actions per
12. (a)
second, with each of its chips having 100 million
Exp.: Recently, Department of Posts, Ministry of transistors. It is located at School of Computer Science
Communications, India and Viet Nara Post of Vietnam
at the University of Manchester.
mutually agreed to issue Postage Stamps on the theme
‘Ancient Architecture’. Stamp depicts Sanchi Stupa of Pratyush
India and Pho Minh Pagoda of Vietnam. The latest supercomputer at Indian Institute of Tropical
Hence, option (a) is correct. Meteorology, Pune ‘Pratyush’ is a Cray-XC40 LC
13. (c) [Liquid Cooled] System with 3315 nodes running Intel
Xeon Broadwell E5-2695 processors with a peak
Exp.: Ambubachi Mela is the four-day long fair celebrated at
performance of 4,006 TFLOPS and a total system
Kamakhya Temple. Kamakhya Temple is atop of the
memory of 414TB. The system is composed of 18
Nilachal Hills. The festival celebration symbolises the
fertility cult of goddess Kamakhya. Hence, option (c) is Compute cabinets and uses Cray’s Aries NOC with
correct. Dragonfly Interconnect network topology. In addition,
the system consists of 16 Intel KNL 7210 accelerator
14. (d)
nodes with a peak performance of 42.56TFLOPS and a
Exp.: Kabir's teachings were based on a complete rejection of total memory of 1.5TB.
major religous traditions. His teachings ridiculed all forms
of idolatry of both Hinduism and Islam. Param Shivay
Hence, statement 1 is NOT correct. National Supercomputing Mission’s first indigenously
zzCompositions of Surdas included Sursagar, Surasaravali built supercomputer ‘Param Shivay’ has been launched
and Sahitya Lahari. Hence, statement 2 is NOT at Indian Institute of Technology, BHU, Varanasi.
correct. Hence, option (b) is correct.

641, First Floor, Dr. Mukherjee Nagar, Delhi-110009 4


Contacts% 011-47532596, (+91) 8448485517
E-mail: drishtiacademy@gmail.com, Web: www.drishtiias.com Facebook: facebook.com/drishtithevisionfoundation
Copyright – Drishti The Vision Foundation
https://telegram.me/pdf4exams https://telegram.me/ias201819

17. (c) Demographic Name, Date of Birth (verified) or Age


Exp.: SHAKTI is an open-source initiative by the RISE group information (declared), Gender, Address, Mobile
at IIT-Madras, which is not only building open source, Number (optional) and Email ID
production grade processors, but also associated (optional), in case of Introducer-
components like interconnect fabrics, verification tools, based enrollment– Introducer name
storage controllers, peripheral IPs and SOC tools. and Introducer’s Aadhaar number, in
The SHAKTI project is a family of 6 processors, based case of Head of Family based
on the RISC-V ISA. It will also develop reference SoCs enrollment– Name of Head of
for each class of processors, which will serve as an Family, Relationship and Head of
exemplar for that family. While the primary focus of the Family’s Aadhaar number; in case of
team is architecture research, these SoCs will be enrollment of child– Enrollment ID
or Aadhaar number of any one
competitive with commercial offerings in the market with
parent, Proof of Relationship (PoR)
respect to area, power and performance.
document
Hence, option (c) is correct.
Biometric Ten Fingerprints, Two Iris Scans, and
18. (d) information Facial Photograph
Exp.: ‘Digital Sky Platform’ is an online IT platform developed zzUIDAI has stated facial recogtition feature as an
for handling applications, permission to fly Remotely additional mode of authentication. Under it live face photo
Piloted Aircrafts (Drones) in India. capture and its verification with the photo obtained ekyc
Hence, option (d) is correct. will be used. Thus benificaries to not need to undergo
Civil Remotely Piloted Aircrafts (RPA) is categorized in additional facial scanning. Hence statement 1 is correct
accordance with Maximum All-Up-Weight (including while 2 is NOT correct.
payload) as indicated below: 20. (c)
i. Nano: Less than or equal to 250 grams.
Exp.: The Telangana State Information Technology, Electronics
Micro: Greater than 250 grams and less than or
ii.
and Communication department (ITE-C) has signed an
equal to 2 kg.
MoU (Memorandum of Understanding) with digital
Small: Greater than 2 kg and less than or equal to
iii. transformation services provider Tech Mahindra to launch
25 kg. India’s first Blockchain District Telangana.
Medium: Greater than 25 kg and less than or equal
iv. The Blockchain District will be a center of excellence for
to 150 kg. Blockchain, an incubator for technology and process
Large: Greater than 150 kg.
v. development with infrastructure and facilities to foster
growth of Indian blockchain start-ups and companies.
19. (a) Tech Mahindra as a founding member of the Blockchain
Exp.: Aadhaar number is a 12-digit random number issued by district will provide platform and technology assistance
the UIDAI (Authority) to the residents of India after to all the incubators in the Blockchain District.
satisfying the verification process laid down by the Hence, option (c) is correct.
Authority. Any individual, irrespective of age and gender, 21. (a)
who is a resident of India, may voluntarily enroll to obtain Exp.: Named after Albert Einstein, who wrote his theory of
Aadhaar number. Person willing to enroll has to provide general relativity in the early 1900s. In it, he suggested
minimal demographic and biometric information during that a massive object (like a galaxy cluster) would warp
the enrollment process which is totally free of cost. An space and time. This process is known today as a
individual needs to enroll for Aadhaar only once and after gravitational lens. When the most massive galaxies and
de-duplication only one Aadhaar shall be generated, as galaxy clusters get in line with a more distant object, they
the uniqueness is achieved through the process of produce an Einstein ring – a type of gravitational lens.
demographic and biometric de-duplication. Einstein rings (and gravitational lenses more generally)
641, First Floor, Dr. Mukherjee Nagar, Delhi-110009 5
Contacts% 011-47532596, (+91) 8448485517
E-mail: helpline@groupdrishti.com, Web: www.drishtiias.com Facebook: facebook.com/drishtithevisionfoundation
Copyright – Drishti The Vision Foundation
https://telegram.me/pdf4exams https://telegram.me/ias201819

give astronomers a huge advantage when they are trying 24. (b)
to look at faraway objects. The rings and lenses magnify Exp.: The Experimental Advanced Superconducting
objects that otherwise would be too distant and dim to Tokamak (East) in Hefei, in China, has been dubbed an
see in today’s telescopes. artificial sun since it replicates the energy-generating
Hence, option (a) is correct.
process of the Sun.
22. (b) In stable fusion, this temperature achieved is one of the
most fundamental elements, because fusion is possible
Exp.: The Bhabha Atomic Reseach Centre (BARC) has
only if the central temperature reaches 100 million
developed a next-generation bulletproof jacket for the degrees.
Indian armed forces, which is not only cheaper but also China’s ‘artificial sun’ has for the first time achieved a
much lighter. plasma central electron temperature of 100 million
Bhabha Kavach, named after nuclear physicist Dr. Homi degrees celsius, marking a key step in China’s future
J. Bhabha, the jacket was developed at BARC’s Trombay fusion reactor experiment, according to the Hefei Institute
centre in response to a request from the Central Reserve of Physical Science under the Chinese Academy of
Police Force (CRPF) and the Ministry of Home Affairs. Sciences.
The jacket is made using extremely hard boron carbide Hence, option (b) is correct.
ceramics that is hot-pressed with carbon nano-tubes and 25. (a)
composite polymer. BARC has been using boron carbide
Exp.: Researchers at Institute for Stem Cell Science &
in the control rods of its nuclear reactors. Regenerative Medicine, Bengaluru, developed poly-
While the cost of a Bhabha Kavach is `70,000, jackets Oxime gel that deactivates toxic chemicals, preventing
of similar strength are available in the range of `1.5 lakh them from going deep into skin, organs.
and have to be imported. Indian farmers usually do not wear any protective gear
Hence, option (b) is correct. while spraying chemicals in fields. This exposes them to
23. (a) harmful toxics contained in pesticides, causing severe
health impacts and even death in extreme cases. Indian
Exp.: Scientists at the Council of Scientific and Industrial scientists have now developed a protective gel to address
Research’s Lucknow-based Indian Institute of Toxicology this problem.
Research (CSIR-IITR) have developed a technology for The gel can be applied on skin and can break down toxic
disinfecting water that promises to provide safe and chemicals in pesticides, insecticides and fungicides
clean drinking water at a cost of just two paise per litre. including the most hazardous and widely used organo
The technology is based on the principle of anodic phosphorous compounds. The gel deactivates these
oxidation. Raw water is passed through a chamber and chemicals, preventing them from going deep into the skin
disinfection occurs with the help of singlet oxygen species and organs like the brain and the lungs. It has been found
generated at the anode. The technology has been named to be effective in tests done in rats and researchers hope
to soon test it in humans.
Oneer – ‘O’ for singlet oxygen species and `neer’ for
Exposure to chemicals contained in pesticides interferes
water.
with an enzyme called acetylcholinesterase (AChE)
The domestic model can provide 10 litres of water in just
which is present in the nervous system and is critical for
five minutes and one full charge of the battery. It is neuromuscular functions. When its functioning is
designed to last four cycles totaling 40 litres. The disrupted by chemical pesticides entering the body
community model, at present, has a capacity of 450 litres through the skin, it can cause neurotoxicity, cognitive
per hour. dysfunction and even death in severe cases. When the gel
The water purifier can eliminate disease-causing was applied on rats and they were exposed to a lethal
pathogens such as virus, bacteria, fungi, protozoa and dose of pesticide MPT, it did not lead to any change in
cyst to provide safe drinking water as per national and their AChE level, showing it could prevent penetration
international standards. of the pesticide into the skin.
Hence, option (a) is correct. Hence, option (a) is correct.

641, First Floor, Dr. Mukherjee Nagar, Delhi-110009 6


Contacts% 011-47532596, (+91) 8448485517
E-mail: drishtiacademy@gmail.com, Web: www.drishtiias.com Facebook: facebook.com/drishtithevisionfoundation
Copyright – Drishti The Vision Foundation
https://telegram.me/pdf4exams https://telegram.me/ias201819

26. (d) zzWeighing about 436 kg, EMISAT based on ISRO’s Indian
Exp.: The government has in 2019 awarded Geographical Mini Satellite -2 (IMS-2) bus platform. The satellite is
Indication (GI) tag to five varieties of Indian coffee intended for electromagnetic spectrum measurement.
including Coorg Arabica. Hence, statements 1 and 2 are correct.
zzThe PS4 orbital platform is envisaged to provide a
zzThe Department for Promotion of Industry and Internal
Trade, under the Ministry of Commerce and Industry, has microgravity environment for research organisations and
recently awarded this tag to — Coorg Arabica coffee academic institutes to perform experiments.
from Karnataka, Wayanad Robusta coffee from Kerala, zzIn this mission, the PS4 hosts three payloads, namely,
Chikmaglur Arabica from Karnataka, Araku Valley Automatic Identification System (AIS) from ISRO,
Arabica from Andhra Pradesh, and Bababudangiris Automatic Packet Repeating System (APRS) from
Arabica coffee from Karnataka. Hence, option (d) is AMSAT (Radio Amateur Satellite Corporation), India
correct. and Advanced Retarding Potential Analyzer for
Ionospheric Studies (ARIS) from Indian Institute of Space
zzIn India, coffee is cultivated in about 4.54 lakh hectare
Science and technology (IIST).
by 3.66 lakh coffee farmers of which 98 per cent are small
farmers. zzThe 28 international customer satellites are from four
countries, viz. Lithuania, Spain, Switzerland and USA.
zzOnce the GI protection is granted, no other producer can
All these satellites are being launched under commercial
misuse the name to market similar products. It also
arrangements.
provides comfort to customers about the authenticity of
that product. 29. (a)
zzSuch products also get premium pricing in the markets. Exp.: Swajal schemes in 115 aspirational districts of the country
27. (c) will involve an outlay of `700 crores through flexi-funds
under the existing National Rural Drinking Water
Exp.: Indian Space Research Organisation has launched a Programme (NRDWP) budget. These schemes will aim
special programme for School Children called ‘Young to provide villages with piped water supply powered by
Scientist Programme’ ‘YUva VIgyani KAryakram’ solar energy. The scheme will train hundreds of rural
from this year, in tune with the Government’s vision ‘Jai technicians for operation and maintenance of Swajal
Vigyan, Jai Anusandhan’. Hence, statements 2 is units. Hence, statement 1 is correct.
correct. Under the scheme, 90% of the project cost will be taken
zzThe Program is primarily aimed at imparting basic care by the Government and the remaining 10% of the
knowledge on Space Technology, Space Science and project cost will be contributed by the community. The
Space Applications to those who have just completed 9th Operations and management of the project will be taken
standard (in the academic year 2018-19) and waiting to care by the local villagers. Hence, statement 2 is NOT
join 10th Std (or just joined 10th Std) with the intent of correct.
arousing their interest in the emerging areas of Space
30. (a)
activities.
Exp.: The Global Innovation Index 2018 (GII) is co-published
zzThe program is thus aimed at creating awareness amongst
by Cornell University, INSEAD, and the World
the School Children who are the future building blocks
Intellectual Property Organization (WIPO, an agency
of our Nation. ISRO has chalked out this programme to
of the United Nations). The core of the GII Report consists
‘Catch them young’.
of a ranking of world economies’ innovation capabilities
Hence, statement 1 is correct.

and results. Hence, option (a) is correct.
28. (c) zzThe Global Innovation Index (GII) aims to capture the
Exp.: India’s Polar Satellite Launch Vehicle (PSLV), in its 47 th
multi-dimensional facets of innovation and provide the
mission (PSLV-C45), will launch EMISAT, the primary tools that can assist in tailoring policies to promote long-
satellite and 28 international customer satellites from term output growth, improved productivity, and job
Satish Dhawan Space Centre (SDSC) SHAR, Sriharikota. growth.

641, First Floor, Dr. Mukherjee Nagar, Delhi-110009 7


Contacts% 011-47532596, (+91) 8448485517
E-mail: helpline@groupdrishti.com, Web: www.drishtiias.com Facebook: facebook.com/drishtithevisionfoundation
Copyright – Drishti The Vision Foundation
https://telegram.me/pdf4exams https://telegram.me/ias201819

zzThe GII helps to create an environment in which with the performance of some of the South Asian
innovation factors are continually evaluated. It provides countries. Nepal moved up from 93 to 84, while Sri Lanka
a key tool and a rich database of detailed metrics for moved up too, from position 80 to 67. Pakistan moved
economies, which in 2018 encompasses 126 economies, from 152 to 151. South Asia experienced the largest
representing 90.8% of the world’s population and 96.3% regional improvement in peacefulness as well as Peace
of global GDP. continues to record a ‘gradual, sustained fall’ across the
The GII relies on two sub-indices world, the report noted. Hence, statement 1 is correct.
zzThe Innovation Input Sub-Index Syria remained the least peaceful country in the world, a
Five input pillars capture elements of the national position that it has held for the past five years.
economy that enable innovative activities: Iceland continues to remain the most peaceful country in
1. Institutions the world, a position it has held since 2008.
2. Human capital and research The IEP, world’s leading think tank that develops metrics
3. Infrastructure to analyse peace and quantify its economic value, released
4. Market sophistication the 12 th edition of the GPI, or measure of global
5. Business sophistication. peacefulness.
zzInnovation Output Sub-Index 32. (c)
Two output pillars capture actual evidence of innovation Exp.: The Organisation for the Prohibition of Chemical
outputs: Weapons (OPCW) is a Hague (Netherland) based
1. Knowledge and technology outputs intergovernmental body that works for the elimination of
2. Creative outputs. chemical weapons. Hence, statement 1 is correct.
Four measures are then calculated: It was formed after the Chemical Weapons Convention
zzInnovation Input Sub-Index: is the simple average of – an arms control treaty that bans the production,
the first five pillar scores stockpiling and use of chemical weapons – entered into
zzInnovation Output Sub-Index is the simple average of force in 1997. The OPCW is the implementing body of
the last two pillar scores the Convention. All its 192 member states are required
zzThe overall GII score is the simple average of the Input to destroy their existing stockpiles of chemical weapons
and Output Sub-Indices and stop large-scale production. Hence, statement 2 is
correct.
zzThe Innovation Efficiency Ratio is the ratio of the Output
The OPCW has the powers to inspect chemical production
Sub-Index over the Input Sub-Index
facilities. Israel has signed the agreement but is yet to
India’s Performance ratify it. The countries that have not signed it are Egypt,
This year, India has moved up 3 places as compared to South Sudan and North Korea.
60th rank in GII last year and emerged as top-ranked 33. (d)
economy in Central and South Asia.
Exp.: In order to promote manufacturing of electric and hybrid
31. (c) vehicle technology and to ensure sustainable growth of
Exp.: According to ‘World Peace Index 2018’ published by the same, Department of Heavy Industry is implementing
Institute for Economics and Peace(IEP) global FAME-India Scheme- Phase-I [Faster Adoption and
peacefulness has deteriorated by 2.38 per cent since 2008. Manufacturing of (Hybrid &) Electric Vehicles in India]
Another important finding in this report is the impact of from 1 April, 2015. Hence, statement 1 is correct.
conflict on macroeconomic performance. In the last 70 The Faster Adoption and Manufacturing of (Hybrid &)
years, per capita growth has been three times higher in Electric Vehicles (FAME-India) Scheme proposes to give
highly peaceful countries when compared to countries a push to electric vehicles (EVs) in public transport and
with low levels of peace. Hence, statement 2 is correct. seeks to encourage adoption of EVs by way of market
India’s rank has marginally improved in ‘global creation and demand aggregation. The draft scheme
peacefulness’, at a time when there is an overall decline envisages the holistic growth of EV industry, including
of global peace owing to escalation of violence in West providing for charging infrastructure, research and
Asia and North Africa. In 2018, when the year 2017 is development of EV technologies and push towards greater
assessed, India’s rank moved up to 136. This is in line indigenization. Hence, statement 2 is correct.

641, First Floor, Dr. Mukherjee Nagar, Delhi-110009 8


Contacts% 011-47532596, (+91) 8448485517
E-mail: drishtiacademy@gmail.com, Web: www.drishtiias.com Facebook: facebook.com/drishtithevisionfoundation
Copyright – Drishti The Vision Foundation
https://telegram.me/pdf4exams https://telegram.me/ias201819

34. (c) Board Bureau has three ex-officio members and three
Exp.: IBSA is a coordinating mechanism amongst three expert members in addition to Chairman. All the Members
emerging countries, three multi ethnic and multicultural and Chairman will be part time. BBB has started to
democracies, which are determined to: function from April 1, 2016.
Hence, option (a) is correct.

zzContribute to the construction of a new international
architecture 37. (c)
zzBring their voice together on global issues Exp.: India was elected to the United Nations’ top human rights
zzDeepen their ties in various areas body for a period of three years from January 1, 2019,
IBSA also opens itself to concrete projects of cooperation getting 188 vote in the Asia-Pacific category, the highest
and partnership with less developed countries. number of votes among all candidates.
The establishment of IBSA was formalised by the Brasilia The 193 member UN General Assembly held elections
Declaration of 6 June, 2003, which mentions India, here for new members to the UN Human Rights Council.
Brazil and South Africa's democratic credentials, their The 18 new members were elected by absolute majority
condition as developing nations and their capacity of through a secret ballot. Countries needed a minimum of
acting on a global scale as the main reasons for the three 97 votes to get elected to the Council.
countries to come together. Their status as middle powers, In the Asia Pacific category, India got 188 votes followed
their common need to address social inequalities within by Fiji with 187 votes, Bangladesh 178, Bahrain and
their borders and the existence of consolidated industrial Philippines 165 each.
areas in the three countries are often mentioned as India had previously been elected to the Geneva-based
additional elements that bring convergence among the Human Rights Council for the 2011-2014 and 2014-2017
members of the Forum. Hence, statement 1 is correct. terms. Hence, statement 1 is correct.
9th IBSA Trilateral Ministerial Commission Meeting The Human Rights Council is an inter-governmental body
held in New York on 27 September, 2018. within the United Nations system responsible for
The Ministers exchanged views on various issues of strengthening the promotion and protection of human
global significance, including peace, security, countering rights around the globe and for addressing situations of
terrorism, Development Cooperation and South-South human rights violations and make recommendations on
Cooperation. The Ministers agreed to continue to them. It has the ability to discuss all thematic human
coordinate on South-South Cooperation in the run up to rights issues and situations that require its attention
BAPA+40 events. Hence, statement 2 is correct. throughout the year. It meets at the UN Office at Geneva.
35. (a) The Council is made up of 47 United Nations Member
Exp.: Ministry of Micro, Small and Medium Enterprises States which are elected by the UN General Assembly.
(MSME) has launched the Mission Solar Charkha for The Human Rights Council replaced the former United
implementation of 50 Solar Charkha Clusters across the Nations Commission on Human Rights. Hence,
country with a budget of `550 crore for the year 2018-19 statement 2 is correct.
and 2019-20. The scheme will generate direct employment 38. (b)
to nearly one lakh persons. Solar Charkha Units have
Exp.: The Asian Infrastructure Investment Bank (AIIB) is a
been classified as village industries. Hence option (a) is
multilateral development bank with a mission to improve
correct.
social and economic outcomes in Asia and beyond.
36. (a) Headquartered in Beijing, it commenced operations in
Exp.: With a view to improve the Governance of Public Sector January 2016 and has now grown to 93 approved
Banks (PSBs), the Government had set up an autonomous members from around the world. By investing in
Banks Board Bureau (BBB). The Bureau recommends sustainable infrastructure and other productive sectors
candidates for their selection as heads of Public Sector today, it will better connect people, services and markets
Banks and Financial Institutions and help Banks in that over time will impact the lives of billions and build
developing strategies and capital raising plans. The Banks a better future. Hence , statement 2 is correct.

641, First Floor, Dr. Mukherjee Nagar, Delhi-110009 9


Contacts% 011-47532596, (+91) 8448485517
E-mail: helpline@groupdrishti.com, Web: www.drishtiias.com Facebook: facebook.com/drishtithevisionfoundation
Copyright – Drishti The Vision Foundation
https://telegram.me/pdf4exams https://telegram.me/ias201819

With the total project portfolios of US$4.4 Billion, India zzTropic of Cancer passes through eight Indian States e.g.,
has been the largest borrower of Asian Infrastructure Gujarat, Rajasthan, Madhya Pradesh, Chhattisgarh,
Investment Bank (AIIB) lending since the time the Bank Jharkhand, West Bengal, Tripura and Mizoram.
started its operations. Hence, option (d) is correct.

Unlike most other multilateral development banks set up 41. (b)
by advanced economies, AIIB is the first major
multilateral development bank where principal Exp.: There are two major island groups in India – one in the
contributors are the borrowing members themselves. Bay of Bengal and the other in the Arabian Sea.
Hence , statement 1 is NOT correct. zzThe entire group of island of Bay of Bengal is divided
into two broad categories – the Andaman in the north and
39. (a)
the Nicobar in the south. They are separated by a water
Exp.: There are two main points of difference between the PMI body which is called the Ten degree channel.
and the IIP. The first is that the PMI is a private sector Hence, statement 1 is NOT correct.
survey while the IIP is gauged by the government. The
zzHowever, some smaller islands are volcanic in origin.
second difference is in what is being measured. While the
Barren island, the only active volcano in India is also
IIP is a measure of output, PMI, as the name suggests,
situated in the Nicobar islands.
measures activity at the purchasing or input stage. Hence,
statement 1 is correct. zzThe coastal line has some coral deposits, and beautiful
Together the two indices provide a composite and beaches. These islands receive convectional rainfall and
reasonably comprehensive information about the formal have an equatorial type of vegetation.
manufacturing sector. As with the IIP, the PMI suffers zzThe islands of the Arabian sea include Lakshadweep and
from the lacuna of not measuring informal sector activity. Minicoy. The entire island group is built of coral deposits.
Hence , statement 2 is NOT correct. There are approximately 36 islands of which 11 are
The Nikkei India Manufacturing PMI is based on data inhabited. Hence, statement 2 is correct.
compiled from monthly survey responses by purchasing zzMinicoy is the largest island with an area of 453 sq. km.
managers in more than 400 manufacturing companies. The entire group of islands is broadly divided by the
The manufacturing sector is divided into eight broad Eleventh degree channel.
categories – basic metals, chemicals and plastics, zzCoral polyps are short-lived microscopic organisms,
electrical and optical, food and drink, mechanical which live in colonies. They flourish in shallow, mud free
engineering, textiles and clothing, timber and paper and and warm waters. They secrete calcium carbonate. The
transport. coral secretion and their skeletons from coral deposits in
PMI is composite index based on five individual indices the form of reefs.
with their own weightages – new orders (weightage 0.3),
zzThey are mainly of three kinds: barrier reef, fringing reef
output (0.25), employment (0.2), suppliers’ delivery times
and atolls.
(0.15), stock of items purchased (0.1) and the delivery
times index inverted so that it moves in a comparable 42. (b)
direction. Once the overall number for the month is Exp.: The Inter Tropical Convergence Zone (ITCZ) is a
computed, the score is arrived at. A score above 50 broad trough of low pressure in equatorial latitudes. This
denotes expansion while one below 50 signifies is where the northeast and the southeast trade winds
contraction. converge. This convergence zone lies more or less
One important advantage the PMI has over the IIP is how
parallel to the equator but moves north or south with the
quickly the data for any reporting period comes out.
apparent movement of the sun. Hence, statement 1 is
40. (d) NOT correct, while 2 is correct.
Exp.: The Tropic of Cancer (23.5 Degree N) passes through zzThe winds that blow from sub-tropical high pressure areas
middle of the Country (India). It parts the country into towards equatorial low pressure areas called trade or
two halves where the part lying south to the Tropic of easterly winds. The trade winds and easterlies are the
Cancer comes to the tropical region, and the part lying permanent winds. These blow constantly throughout the
north to it falls in the subtropical region. year in a particular direction.

641, First Floor, Dr. Mukherjee Nagar, Delhi-110009 10


Contacts% 011-47532596, (+91) 8448485517
E-mail: drishtiacademy@gmail.com, Web: www.drishtiias.com Facebook: facebook.com/drishtithevisionfoundation
Copyright – Drishti The Vision Foundation
https://telegram.me/pdf4exams https://telegram.me/ias201819

43. (a) 45. (a)


Exp.: EI-Nino is a complex weather system that appears once Exp.: Operation Nistar
every three to seven years, bringing drought, floods and Under it thirty-eight Indians were brought back home by
other weather extremes to different parts of the world.
the Indian Navy, four days after they were rescued from
zzThe system involves oceanic and atmospheric phenomena
the cyclone-hit Socotra island in Yemen (cyclone
with the appearance of warm currents off the coast of Peru
in the Eastern Pacific and affects weather in many places Mekunu). Indian naval ship INS Sunayna carried out the
including India. operation.
zzEI-Nino is merely an extension of the warm equatorial Operation Raahat
current which gets replaced temporarily by cold Peruvian Conducted to evacuate almost 5,000 Indians and nearly
current or Humboldt current.
1,000 citizens from 41 other countries from war ravaged
zzThis current increases the temperature of water on the
Yemen.
Peruvian coast by 10°C.
Hence, option (a) is correct.
zzThis results in:
„„The distortion of equatorial atmospheric circulation; 46. (d)
„„Irregularities in the evaporation of sea water;
Exp.: The Indus also known as the Sindhu, is the westernmost
„„Reduction in the amount of planktons which further of the Himalayan rivers in India. It originates from a
reduces the number of fish in the sea.
glacier near Bokhar Chu (31°15' N latitude and 81°40' E
The word EI-Nino means ‘Child Christ’ because this
longitude) in the Tibetan region at an altitude of 4,164 m
current appears around Christmas in December. December
is a summer month in Peru (Southern Hemisphere). EI- in the Kailash Mountain range. In Tibet, it is known as
Nino is used in India for forecasting long range monsoon ‘Singi Khamban; or Lion’s mouth. After flowing in the
rainfall. northwest direction between the Ladakh and Zaskar
Hence, option (a) is correct. ranges, it passes through Ladakh and Baltistan. The Indus
44. (a) receives a number of Himalayan tributaries such as the
Exp.: Par Tapi Narmada Link proposes to transfer water from Shyok, the Gilgit, the Zaskar, the Hunza, the Nubra, the
the water surplus regions of Western Ghats to the water Shigar, the Gasting,the kabul and the Dras. Hence, pair
deficit regions of Saurashtra and Kutch. Hence, statement 1 is correctly matched.
1 is correct. zzGanga rises in the Gangotri glacier near Gaumukh
The link project includes seven reservoirs proposed in
(3,900 m) in the Uttarkashi district of Uttarakhand.The
north Maharashtra and south Gujarat. The water from the
seven proposed reservoirs would be taken through a 395 Mahananda is another important tributary of the Ganga
km. long canal including the 33km. length of the feeder rising in the Darjiling hills. It joins the Ganga as its last
canals to take over a part of the command of the on-going left bank tributary in West Bengal. Hence, pair 2 is
Sardar Sarovar Project, while irrigating small enroute correctly matched.
areas. Hence statement 2 is NOT correct.
zzThe Yamuna, the western most and the longest tributary
This would save Sardar Sarovar water which will be used
to extend irrigation in Saurashtra and Kutch region. The of the Ganga, has its source in the Yamunotri glacier on
link mainly envisages construction of seven dams, three the western slopes of Banderpunch range (6,316 km.). It
diversion weirs, two tunnels (5.0 km. & 0.5 km. of length), joins the Ganga at Prayag (Allahabad). The Chambal
395 km. long canal (205 km. in Par-Tapi portion including rises near Mhow in the Malwa plateau of Madhya Pradesh
the length of feeder canals and 190 km in Tapi-Narmada and flows northwards through a gorge up wards of Kota
portion), 6 power houses and a number of cross-drainage
in Rajasthan, where the Gandhisagar dam has been
works. Besides providing irrigation benefits to the enroute
command and Narmada command, the link will generate constructed. From Kota, it traverses down to Bundi, Sawai
hydropower of the order of 93.00 Mkwh through the Madhopur and Dholpur, and finally joins the Yamuna.
power houses installed at four dam sites Hence, pair 3 is correctly matched.

641, First Floor, Dr. Mukherjee Nagar, Delhi-110009 11


Contacts% 011-47532596, (+91) 8448485517
E-mail: helpline@groupdrishti.com, Web: www.drishtiias.com Facebook: facebook.com/drishtithevisionfoundation
Copyright – Drishti The Vision Foundation
https://telegram.me/pdf4exams https://telegram.me/ias201819

47. (a) zzThese plains have characteristic features of mature stage


Exp.: Black soil covers most of the Deccan Plateau which of fluvial erosional and depositional landforms such as
includes parts of Maharashtra, Madhya Pradesh, Gujarat, sand bars, meanders, oxbow lakes and braided channels.
Andhra Pradesh and some parts of Tamil Nadu. Western 49. (c)
part of the Deccan Plateau, the black soil is very deep.
Exp.: PMMVY is a Maternity Benefit Programme that is
These soils are also known as the ‘Regur Soil’ or the
implemented in all the districts of the country. Hence,
‘Black Cotton Soil’. Hence, statement 1 is correct.
statement 1 is correct.
zzThe black soils are generally clayey, deep and impermeable. The scheme has re-designed the transfer of funds process
They swell and become sticky when wet and shrink when to bring in efficiency, effectiveness and improve service
dried. So, during the dry season, these soil develop wide delivery.
cracks. Thus, there occurs a kind of ‘self ploughing’. PMMVY-CAS collates the list of beneficiaries for the
Because of this character of slow absorption and loss of whole State/UT and sends it to SNO electronically in
moisture, the black soil retains the moisture for a very form of a batch file. Hence, only the SNO interacts with
long time, which helps the crops, especially, the rain fed
PFMS using his Digital Signature Certificate, with the
ones, to sustain even during the dry season.
approval facilitated by PMMVY-CAS. PMMVY involves
zzChemically, the black soils are rich in lime, iron, magnesia transferring cash benefits to beneficiaries through DBT
and alumina. They also contain potash. Hence, statement mode only. Hence, statement 2 is correct.
2 is NOT correct.
50. (b)
zzBut these soils lack in phosphorous, nitrogen and organic
matter. The colour of the soil ranges from deep black to Exp.: Govt. of India approved a new program titled Global
grey. Hence, statement 3 is NOT correct. Initiative of Academic Networks (GIAN). The programme
aims at tapping the talent pool of scientists and
48. (d)
entrepreneurs, internationally to encourage their
Exp.: The northern plains are formed by the alluvial deposits engagement with the institutes of Higher Education in
brought by the rivers – the Indus, the Ganga and the India so as to augment the country's existing academic
Brahmaputra. Hence, statement 1 is NOT correct. resources, accelerate the pace of quality reform, and
zzThese plains extend approximately 3,200 km. from the elevate India’s scientific and technological capacity to
east to the west. The average width of these plains varies global excellence. Hence, option (b) is correct.
between 150–300 km. The maximum depth of alluvium
51. (b)
deposits varies between 1,000–2,000 m. From the north
to the south, these can be divided into three major zones: Exp.: The Annapurna Scheme is being implemented by the
the Bhabar, the Tarai and the alluvial plains. The alluvial Ministry of Rural Development. Hence, statement 1 is
plains can be further divided into the Khadar and the NOT correct.
Bhangar. Bhabar is a narrow belt ranging between 8-10 zzUnder Annapurna Scheme food grains are provided
km parallel to the Shiwalik foothills at the break-up of through State Governments/ Union Territory (UT)
the slope. As a result of this, the streams and rivers coming Administrations to indigent senior citizens of 65 years of
from the mountains deposit heavy materials of rocks and age or above who are not getting old age pension under
boulders, and at times, disappear in this zone. South of the National Old Age Pension Scheme(NOAPS). Hence,
the Bhabar is the Tarai belt, with an approximate width statement 2 is correct.
of 10–20 km. where most of the streams and rivers re- zzFood grains are provided at 10 kg per person per month
emerge without having any properly demarcated channel, free of cost to the beneficiaries.
thereby, creating marshy and swampy conditions known
as the Tarai. This has a luxurious growth of natural 52. (b)
vegetation and houses a varied wildlife. Exp.: Rajatarangini: It is a historical chronicle of early
zzThe south of Tarai is a belt consisting of old and new India, written in Sanskrit verse by the Kashmiri Brahman
alluvial deposits known as the Bhangar and Khadar Kalhana in 1148, that is justifiably considered to be the
respectively. Hence, statement 2 is NOT correct. best and most authentic work of its kind. It covers the

641, First Floor, Dr. Mukherjee Nagar, Delhi-110009 12


Contacts% 011-47532596, (+91) 8448485517
E-mail: drishtiacademy@gmail.com, Web: www.drishtiias.com Facebook: facebook.com/drishtithevisionfoundation
Copyright – Drishti The Vision Foundation
https://telegram.me/pdf4exams https://telegram.me/ias201819

entire span of history in the Kashmir region from the 55. (a)
earliest times to the date of its composition. Hence, pair Exp.: One of the remarkable features of the Chola administration
1 is NOT correctly matched. was their encouragement to local self-government in the
Lilawati: It is Indian mathematician Bhaskara II’s
villages all over their empire. There were two kind of
treatise on mathematics, written in 1150. It is the first assemblies, called the ‘ur’ and the ‘sabha’ or ‘mahasabha’.
volume of his main work, the Siddhānta Shiromani, The ‘ur’ was a general assembly of the village.
alongside the Bijaganita, the Grahaganita and the ‘Mahasabhas’ was the gathering of the adult men in the
brahmana villages called ‘agraharas’. These villages
Goladhyaya. Hence, pair 2 is correctly matched.
enjoyed a large measure of autonomy. Hence, statement,
Kavirajamarga: It is the earliest available work on
1 is correct.
rhetoric, poetics and grammar inthe Kannada zzThe affairs of the brahmana village were manged by an
language. It was inspired by or written in part by the executive committee to which educated person owning
famous Rashtrakuta King Amoghavarsha. Hence, pair 3 property were elected either by drawing lots or by rotation.
is correctly matched. These members had to retire every three years. There
were other committees for helping in the assessment and
53. (b)
collection of land revenue for maintenance of law and
Exp.: In the Deccan during medieval times (Rashtrakutas order, justice, etc. One of the important committees was
period), there were hereditary revenue officers called as the tank committee which looked after the distribution of
water to the fields.
‘nad-gavundas’ or ‘desa-gramakutas’. They discharged
zzThe mahasabhas could settle new lands, and exercise
same functions as the Deshmukhs and Deshpandes of
ownership rights over them. It could also raise loans for
later times in Maharashtra. As their power grew, the
the village and levy taxes.
village committees became weaker. The central ruler also Hence, statement 2 is NOT correct.
found it difficult to assert their authority over them. zzThe self-government enjoyed by these Chola villages was
Hence, option (b) is correct. a very fine system. To some extent this system worked in
54. (d) other villages as well. However, the growth of local
intermediaries tended to restrict their autonomy.
Exp.: According to Tibetan chronicles, the Pala rulers (of
56. (a)
Bengal region) were great patrons of Buddhist learning
and religion. The Nalanda university which had been Exp.: A number of popular saints called Nayanars and Alvars
who were devotees of Shiva and Vishnu respectively.
famous all over the eastern world was revived by
They flourished in the Tamil area between the sixth and
Dharmapala (ruled 8th century) and 200 villages were ninth centuries. They composed their works in Tamil
set apart for meeting its expenses. He founded the and other languages of the area. Hence, Statement 1 is
Vikramshila university. Hence, statement 1 is correct. NOT correct.
The real founder of the Gurjara-Pratihars empire and the Statement 2 is correct. These saints rejected austerities.
greatest ruler of the dynasty was Bhoja (or Mihir Bhoja). They looked upon religion not as a matter of cold, formal
worship but as a living bond based on love between the
He was a devotee of Vishnu and adopted the title of
God and the worshipper. These saints went from place to
‘Adivaraha’ which has been found inscribed in some of place carrying their message of love and devotion. Many
his coins. Hence, statement 2 is correct. of them belonged to the lower classes. There was also a
Amoghavarsha (814-878 CE) was a ruler of Rashtrakutas woman saint, Andal (Alvars). Hence, statement 2 is
empire. He ruled for 68 years, he preferred the pursuit of correct.
religion and literature to war. He himself was an author Nayanars were group of 63 saints devoted to Shiva, whose
and is credited with writing the first kannada book on writings collected into 11 volumes under the name
Tirumurais in the early part of the twelfth century and are
poetics. He was a great builder and is said to have built
looked upon the fifth veda. Later in twelth century one
the capital city Manyakhet. He is said to have been a Jain more volume was added which was called as Periya
but he also patronised other faiths. Puranam.

641, First Floor, Dr. Mukherjee Nagar, Delhi-110009 13


Contacts% 011-47532596, (+91) 8448485517
E-mail: helpline@groupdrishti.com, Web: www.drishtiias.com Facebook: facebook.com/drishtithevisionfoundation
Copyright – Drishti The Vision Foundation
https://telegram.me/pdf4exams https://telegram.me/ias201819

Alvars were group of 12 saints devoted to Vishnu first battle, the Ghuri forces were completely routed,
flourished between 5th century and 10th century. Andal is Muizzuddin Muhammad’s life being saved by a young
the only female saint among them. The collection of their Kahlji horseman. However, in the second battle of Tarain
hymns is known as DivyaPrabandha. (1192), Prithviraja Chauhan was defeated by Muizzuddin
57. (c) Muhammad with more careful preparation of huge
standing army. Hence, statement 1 is correct.
Exp.: In the late 12th century and early 13th century, leading
states of North India suffered defeat within short span of 59. (c)
about 15 years by the Turkish armies. Exp.: Balban who ruled India as the Sultan of Delhi from 1265
Reasons to 1287 CE was one of the greatest Sultans of the medieval
period. He played the role of kingmaker. He held the
zzThe Indians had the advantage of elephants, but the Turks’
position of naib or deputy to Nasiruddin Mahmud, a
horses were swifter and sturdier than the horses imported
younger son of Iltutmish, whom Balban had helped in
into India. Hence, statement 1 is correct.
securing the throne in 1246. Later, with diplomacy and
zzThe superiority of the Turks was more social and force usurped the throne in 1265 CE. Hence, Statement
organisational. The growth of feudalism had weakened 1 is correct.
the administrative structure and military organisation of He refused to laugh and joke in the court and even gave
the Indian states. The rulers had to depend more on the up drinking wine so that no one may see him in a non-
various chiefs who rarely acted in coordination and serious mood. To emphasize that the nobles were not his
quickly dispersed to their areas after battle. On the other equlas, he insisted on the ceremony of sijida and paibos
hand, the tribal structure of the Turks and the growth of (prostration and kissing the monarch’s feet). Hence,
iqta and khalisa systems enabled the Turks to maintain statement 2 is correct.
large standing armies which could be kept in the field for To deal with internal disturbances and to repel Mongols
a long time. Hence, statement 2 iscorrect. who had entrenched themselves in the Punjab and posed
zzThe Rajputs put up a stout and prolonged resistance to a serious danger to the Delhi Sultanat, he need a strong
the incursions of Arabs and Turks, but at no time did they personalised army. For the purpose, he reorganised the
take the offensive and try to push the Arabs or Turks from military department (diwan-i-arz) and pensioned off those
the strategic lands they occupied. Thus, they allowed the soldiers and troopers who were no longer fit for service.
frontier areas of Indi, especially Afghanistan and Punjab, 60. (c)
which were the outer bastion of the Ganges heartland to
remain in the hostile hands. Exp.: BHARAT-22 Exchange Traded Fund (ETF) managed by
ICICI Prudential Mutual Fund. Hence, statement 2 is
58. (a) correct.
Exp.: Mahmud (971-1030 CE) ascended the throne (998 CE) zzThe strength of this ETF lies in the specially created Index
at Ghazni. He played an important role in the defence of S&P BSE BHARAT-22 INDEX.
the Islamic states against the Turkish tribes and in the zzThis Index is a unique blend of shares of key CPSEs,Public
Iranian cultural renaissance. However, in India his Sector Banks (PSBs) and also the Government owned
memory is only that of a plunderer and a destroyer of shares in blue chip private companies like Larsen & Tubro
temples. He is said to have made 17 raids into India. The (L&T), Axis Bank and ITC. Hence, statement 1 is
initial raids were directed against the Hindushahi rulers correct.
who at time held Peshawar and the Punjab. He also fought The shares of the Government companies represent 6 core
against the Muslim rulers of Multan who belonged to sectors of the economy - Finance, Industry, Energy,
Muslim sect. No attempt was made by Mahmud to annex Utilities, Fast Moving Consumer Goods (FMCG) and
any of the plundered area, although they captured Punjab Basic Materials.
and Multan. Hence, statement 2 is NOT correct. zzThis combination makes the Index broad-based and
zzThe first battle of Tarain (1191) was happened between diversified. The Sector and Stock exposure limits help in
Muizzuddin Muhammad and Prithiviraja Chauhan. The risk management and reduction of concentration,
conflict started with rival claims for Tabarhinda. In the providing stability to the Index.
641, First Floor, Dr. Mukherjee Nagar, Delhi-110009 14
Contacts% 011-47532596, (+91) 8448485517
E-mail: drishtiacademy@gmail.com, Web: www.drishtiias.com Facebook: facebook.com/drishtithevisionfoundation
Copyright – Drishti The Vision Foundation
https://telegram.me/pdf4exams https://telegram.me/ias201819

zzThe strength of the Index has been demonstrated in its 63. (c)
performance from the time of its launch in August 2017 Exp.: Pandita Ramabai
wherein it has out-performed the NIFTY-50 and Sensex. She was a great scholar of Sanskrit. She wrote a book
zzThe Index constituents include leading Maharatanas and about the miserable lives of upper-caste Hindu women.
Navratanas such as Coal India, GAIL, Power Grid zzShe founded a widows’ home at Poona to provide shelter
Corporation of India Ltd. (PGCIL), National Thermal to widows who had been treated badly by their husbands’
Power Corporation (NTPC), Indian Oil Corporation Ltd., relatives. This was one of the country’s first women’s
Oil & Natural Gas Corporation (ONGC), Bharat shelters in the country.
Petroleum, and National Aluminum Company (NALCO),
zzWomen were trained in the shelter so that they could
three Public Sector Banks such as SBI, Bank of Baroda
support themselves economically. She travelled around
apart from the 3 private sector companies mentioned
India in the 19th century to give lectures on women’s
earlier.
emancipation. Hence, option (c) is correct.
61. (b)
64. (c)
Exp.: Firuz Tughlaq tried to win over the theologians by
Exp.: Swami Dayanand Saraswati founded the Arya Samaj in
proclaiming that he was a true Muslim King and that the
1875. It was an organisation that attempted to reform
state under him was a truly Islamic state. He gave number
Hinduism. Hence, statement 1 is correct.
of concessions to the theologians. He tried to ban practices
zzThe Arya Samaj was one of the most dynamic socio-
which the orthodox theologians considered as un-Islamic.
He made jizyah a separate tax, earlier it was a part of land religious reform movement modem society has ever seen.
revenue. He refused to exempt the brahamanas from It intended to have a national scope and a program of
the payment of jizyah since this was not provided for reconstruction in the social and religious spheres.
in the Sharia. Hence, statement 1 is NOT correct. zzThough the Arya Samaj was primarily concerned with
AlauddinKhalji’s measures to control the markets was social and religious reforms, its political impact was most
one of the great wonders of the world during his conspicuous.
contemporary times. He sought to fix the cost of all zzThe Arya Samaj movement was begun to revive the study
commodities from foodgrains, sugar and cooking oil to of the Vedas and to worship one God. Hence, statement
needles, and from costly imported cloth to horses, cattle 2 is correct.
and slave boys and girls. For the purpose, he set up three zzDayanand defined Aryas as those who are true in word,
markets at Delhi– one for foodgrains, second for costly deed and thought, promote public good and are learned.
cloth and the third for horses, slaves and cattle. Each
65. (a)
market was under the control of a high officer called
shahna, who maintained a register of the merchants and Exp.: Disorder Inquiry Committee, which came to be more
strictly controlled the shopkeeper and prices. popularly known as Hunter Committee, was headed by
Hence, statement 2 is correct. Lord William Hunter. Hence, statement 2 is NOT
correct.
62. (d)
zzIt was constituted by an order of the then Secretary Of
Exp.: According to tradition and epigraphic evidence two State of India, Edwin Montagu to investigate the
brothers, Harihara and Bukka, founded the Vijayanagara disturbance caused by Jallianwala Bagh massacre in
Empire in 1336. Bombay, Delhi and Punjab. Hence, statement 1 is
zzDuring this period, warfare depended upon effective correct.
cavalry, the import of horses from Arabia and Central zzIt also aimed to find the cause, and the measures that could
Asia was very important for rival kingdoms. be taken to cope with these disturbance.
zzThis trade was initially controlled by Arab traders. Local
66. (b)
communities of merchants known as kudirai chettis or
horse merchants also participated in these exchanges. Exp.: Based on the Montagu Chelmsford Reforms, the
Hence, option (d) is correct.
Government of India Act, 1919 was enacted.

641, First Floor, Dr. Mukherjee Nagar, Delhi-110009 15


Contacts% 011-47532596, (+91) 8448485517
E-mail: helpline@groupdrishti.com, Web: www.drishtiias.com Facebook: facebook.com/drishtithevisionfoundation
Copyright – Drishti The Vision Foundation
https://telegram.me/pdf4exams https://telegram.me/ias201819

zzThe Act introduced dyarchy (rule of executive councillor zzIt separated, for the first time, provincial budgets from
and popular minister) for the executive at the level of the the Central budget and authorised the provincial
provincial government. Hence, option (b) is correct. legislatures to enact their budgets.
zzSubjects were divided into two lists- reserved (law and zzIt provided for the appointment of a statutory commission
order, finance, land, revenue, irrigation etc.) and to inquire into and report on its working after ten years
transferred (education, health, local government, Industry, of its coming into force.
agriculture, excise etc.). Hence, option (d) is correct.

67. (a) 69. (d)

Exp.: Being inspired by the reading of John Ruskin’s ‘Unto Exp.: After the failure of the Cripps Mission, Mahatma Gandhi
This Last’ Mahatma Gandhi set up Phoenix Farm in 1904 launched his major movement against British rule called
on the north-western edge of Durban to experiment with ‘Quit India’ campaign, which began in August 1942.
satyagraha, sarvodaya and ahimsa (non-violence). zzAlthough Gandhiji was jailed at once, younger activists
zzIt was here Gandhiji first used three principles to
organised strikes and acts of sabotage all over the country.
champion the cause of Mining and sugarcane workers, zzParticularly active in the underground resistance were
liberation of women and fight against alcohol. socialist members of the Congress, such as Jayaprakash
Narayan.
zzThe Phoenix Settlement was set up to put into practice
the values which he followed and preached for the rest zzIn several districts, such as Satara in the west and
of his life – self-help, dignity of labour and simplicity. Medinipur in the east, ‘independent’ governments were
proclaimed. Chittu Pandey formed a government in Balia
Hence, option (a) is correct.
while YB Chavan and Nana Patil in Satara.
68. (d)
zzQuit India movement was unique in the sense that it saw
Exp.: The Government of India Act, 1919 was enacted women participation where they not only participated as
following the recommendations of Montagu Chelmsford equals but also led the movement.
Reforms. zzThe movement saw rise of leaders like, Dr. Ram Manohar
Following were the features of Government of India Act, Lohia, Jai Prakash Narayan and Aruna Asaf Ali. There
1919– was Matangini Hazra, who lead a procession of 6,000
zzIt introduced, for the first time, bicameralism and direct people, mostly women, to ransack a local police station.
elections in the country. Thus, the Indian Legislative zzThen there was Sucheta Kripalani who later went on to
Council was replaced by a bicameral legislature consisting become first women Chief Minister of India.
of an Upper House (Council of State) and a Lower House zzOrissa had Nandini Devi and Sashibala Devi while Assam
(Legislative Assembly). saw participation of young girls like Kanaklata Baruah
zzIt required that the three of the six members of the and Kahuli Devi who died of police atrocities. Contribution
Viceroy’s executive Council (other than the commander- of Usha Mehta was unique as she started a Secret
in-chief) were to be Indian. Congress Radio in Mumbai.
zzIt introduced dyarchy for the executive at the level of the 70. (a)
provincial government. Exp.: The Home rule League was inspired by the Irish Home
zzIt extended the principle of communal representation Rule League. It aimed at achieving self government for
by providing separate electorates for Sikhs, Indian India through constitutional means. The idea was
Christians, Anglo-Indians and Europeans. popularised by Annie Besant, the president of the
zzIt granted franchise to a limited number of people on international Theosophical society. Hence, statement 1
the basis of property, tax or education. is correct, while statement 2 is NOT correct.
The programme of the league declared by her included:
zzIt provided for the establishment of a public service
commission. Hence, a Central Public Service Commission zzReligious liberty
was set up in 1926 for recruiting civil Servants. zzNational Education

641, First Floor, Dr. Mukherjee Nagar, Delhi-110009 16


Contacts% 011-47532596, (+91) 8448485517
E-mail: drishtiacademy@gmail.com, Web: www.drishtiias.com Facebook: facebook.com/drishtithevisionfoundation
Copyright – Drishti The Vision Foundation
https://telegram.me/pdf4exams https://telegram.me/ias201819

zzSocial Reforms British Columbia. The 376 passengers were not allowed
zzPolitical Reforms to disembark in Canada and were sent back to India. They
Annie Besant formally inaugurated the Home Rule arrived in Calcutta two months later were they were fired
League in Sept. 1916. upon by the British and 19 of the passengers died.
Bal Gangadhar Tilak also founded an alternative Indian Canadian Prime Minister Justin Trudeau has apologised
Home Rule League in April 1916 at Poona. for the incident in the Canadian Parliament. Hence,
Tilak focussed on Maharashtra and Central Provinces, option (a) is correct.
while Annie besant focussed on the rest of India. 73. (b)
The goal of both the leagues was the same i.e. to attain
Exp.: The Parliamentary Government also known as Cabinet
self government or Home Rule within the British Empire.
government or responsible government or Westminster
71. (b) Model of government is prevalent in Britain, Japan,
Exp.: The suspension of Non-cooperation movement and the Canada, India etc. Its main features are written below:
imprisonment of Gandhi created a vacuum in the zzNominal and Real Executive: The President acts as a
congress’s plan of action. When the Congress met at Gaya nominal executive, while the Prime Minister acts as the
in 1922, there emerged two opposing groups: real executive.The President, thus, is the head of State,
whereas the Prime Minister is the head of government.
The Pro Changers Hence, statement 1 is NOT correct.
They were headed by C.R. Das, Vithalbhai Patel, Hakim
zzCollective Responsibility: The ministers are collectively
Ajmal Khan and Motilal Nehru.
responsible to the Parliament in general. Hence,
They were in favour of contesting the council elections
statement 2 is correct.
which were to be held in 1923. The Pro changers formed
„„Majority Party Rule
the Swaraj Party following the government of India act
„„Double Membership (ministers are members of both
of 1919. Hence, statement 1 is correct while statement
the legislature and executive)
2 is NOT correct.
„„Leadership of Prime Minister
The No Changers
They included leaders like Ansari, Rajagopalachari, „„Dissolution of Lok Sabha

Kasturiranga Iyengar, Rajendra Prasad and Vallabhbhai „„Political Homogeneity


Patel. 74. (b)
They wanted to follow constructive rural work and not
take part in Council elections. Exp.: The President of India is elected by indirect election
The Swaraj Party did not disassociate itself from the along with the system of proportional representation by
congress but claimed to be its integral part. means of the single transferable vote.
Article 54 of the Indian Constitution prescribes for an
Due to their efforts the government was forced to grant
electoral college for the election of the President
protection to Tata Iron and Steel. They were also able to
consisting–
secure economic benefits like the abolition of excise duty
on cotton, reduction of duty on salt, improvement in the zzThe elected members of Lok Sabha
condition of labour, certain protections for trade unions, zzThe elected members of Rajya Sabha
etc. Hence, statement 3 is correct. zzThe elected members of Legislative Assemblies of
States
72. (a)
zzThe elected members of Legislative Assemblies of
Exp.: Komagata Maru was a Japanese Ship hired by Gurdit Union Territories of Delhi and Puducherry
Singh to transport Indians from various parts of Asia to Hence, option (b) is correct.
Canada in 1914.It set sail from Hong Kong on April 4,
75. (d)
1914, with 165 passengers; more passengers joined in
Shanghai and Yokohama. The ship continued its journey Exp.: Article 63 states that there shall be a Vice-President of
to the western coast of Canada with 376 passengers. After India. He shall be ex-officio Chairman of the Council of
about 40 days, on May 23, it arrived on the shores of States (Article 64).

641, First Floor, Dr. Mukherjee Nagar, Delhi-110009 17


Contacts% 011-47532596, (+91) 8448485517
E-mail: helpline@groupdrishti.com, Web: www.drishtiias.com Facebook: facebook.com/drishtithevisionfoundation
Copyright – Drishti The Vision Foundation
https://telegram.me/pdf4exams https://telegram.me/ias201819

zzArticle 67 a Vice-President may be removed from his 78. (b)


office by a resolution of the Council of States passed by Exp.: Article 48 (Organisation of agriculture and animal
a majority of all the then members of the Council and husbandry): The State shall endeavour to organise
agreed to by the House of the People; but no formal agriculture and animal husbandry on modern and
impeachment is required for his removal. Hence, scientific lines and shall, in particular, take steps for
statement 1 is NOT correct. preserving and improving the breeds, and prohibiting the
zzIn case of vacancy of the post of President, being caused slaughter, of cows and calves and other milch and draught
by the expiration of the term of the sitting President, an cattle.
election to fill the vacancy must be completed before the Hence, (b) is the correct answer.
expiration of the term (Article 62(1)). Therefore, Government both Central and State
zzBut in order to prevent an ‘interregnum’, owing to any government has taken slew of measures Ex: Rashtriya
possible delay in such completion, it is provided that the Gokul Mission, National Programme for Bovine
outgoing president must continue to hold office, until his Breeding, PashuSanjivni, etc.
successor enters upon his office.(Article 56(1) (c)). 79. (c)
zzThere is no scope for the Vice-President getting a chance
Exp.: The Directives Principles have been criticised mainly
to act as President in this case. Hence, statement 2 is
because of their non-justiciable character. Therefore, the
NOT correct.
courts cannot declare a law violative of any of the
76. (c) Directive Principles as unconstitutional and invalid.
Exp.: The Supreme Court (Article 32) as well as the High However, they can uphold the validity of a law on the
Courts (article 226) are empowered to issue five different ground that it was enacted to give effect to a directive.
kinds of writs in case of the violation of the fundamental Hence, statement 1 is correct.
rights of an individual or the other. In the Minerva Mills case (1980), the Supreme Court held
that the Indian Constitution is founded on the bedrock of
zzThe Parliament (under Article 32) can empower any other
the balance between the Fundamental Rights and the
court to issue these writs. But, since no such provision
Directive Principles. They together constitute the core of
has been made so far, hence, no courts other than the
commitment to social revolution. The present position is
Supreme Court and the High Court have got the power
that the Fundamental Rights enjoy supremacy over the
to issue writs. Hence, statement 1 is correct.
Directive Principles. Yet, this does not mean that the
zzHabeas Corpus: The word ‘Habeas Corpus’ means ‘to Directive Principles cannot be implemented. The
have the body of’. Through this writ, the Court secures Parliament can amend the Fundamental Rights for
the body of a person who has been imprisoned and to implementing the Directive Principles, so long as the
obtain knowledge of the reason of his detention. amendment does not damage or destroy the basic structure
zzThis writ can be issued against any person either the of the Constitution. Hence, statement 2 is correct.
official or the private individual who has detained any
80. (b)
person in his custody. Hence, statement 2 is correct.
Exp.: The Central Information Commission (CIC) was
77. (c)
established under the provisions of the Right to
Exp.: Sardar Swaran Singh Committee:In 1976, the Congress Information Act (2005). It falls under the Ministry of
Party set up the Sardar Swaran Singh Committee to make Personnel, Public Grievances and Pensions. Hence,
recommendations about fundamental duties, the need statement 1 is correct.
and necessity of which was felt during the operation of Section 12 (5) of the RTI Act provides for selecting people
the internal emergency (1975–1977). The committee of eminence in public life with wide knowledge and
recommended the inclusion of a separate chapter on experience in law, science and technology, social service,
fundamental duties in the Constitution. Hence, option management, journalism, mass media or administration
(c) is correct. and governance.

641, First Floor, Dr. Mukherjee Nagar, Delhi-110009 18


Contacts% 011-47532596, (+91) 8448485517
E-mail: drishtiacademy@gmail.com, Web: www.drishtiias.com Facebook: facebook.com/drishtithevisionfoundation
Copyright – Drishti The Vision Foundation
https://telegram.me/pdf4exams https://telegram.me/ias201819

They should not be a Member of Parliament or Member The court also held that the right to life under Article 21
of the Legislature of any State or Union Territory. They of Indian Constitution would include the right of
should not hold any other office of profit or connected witnesses to testify freely in courts.The right to life
with any political party or carrying on any business or guaranteed to the people of this country also includes in
pursuing any profession. Hence, statement 2 is NOT its fold the right to live in a society, which is free from
crime and fear, and the right of witnesses to testify in
correct.
courts without fear or pressure. Hence, statement 1 is
Value Addition: It has been alleged that government
correct.
selects a greater number of former bureaucrats for these
posts, thus diluting the letter and spirit of theSection 12(5) Key features of the Scheme
of the RTI Act. zzIt identifies three categories of witnesses as per threat
Recently, Government has appointed Sudhir Bhargava as perception:
new Chief Information Commissioner along with four zzCategory A: Those cases where threat extends to life of
new Information Commissioners in the Central witness or family members during investigation, trial or
Information Commission (CIC). even thereafter.
zzCategory B: Those cases where the threat extends to
81. (a) safety, reputation or property of the witness or family
Exp.: Schedule VII of Indian Constitution consists of subjects/ members during the investigation or trial.
powers divided in terms of List I (Union List), List II zzCategory C: Cases where the threat is moderate and
(State List) and List III (Concurrent List). It givens the extends to harassment or intimidation of the witness or
Indian polity its federal character i.e division of power his family members, reputation or property during the
between Central Government and State Government of investigation, trial or thereafter.
Indian political setup. Hence, statement 1 is correct. The expenses for the programme will be met from a
Entry 56 of List I of Seventh Schedule provides that Witness Protection Fund to be established by states and
Union Territories. This fund will have annual budgetary
‘Regulation and development of inter-State rivers and
allocation and will also accept the donations from
river valleys to the extent to which such regulation and
philanthropic organisations. Hence, statement 2 is
development under the control of the Union is declared correct.
by Parliament by law to be expedient in the public The application for protection will have to be filed before
interest’. the ‘Competent Authority’ along with supporting
Entry 17 under list II of seventh schedule provides that documents. The Authority will in turn seek a ‘Threat
‘Water, that is to say, water supplies, irrigation and canals, Analysis Report’ from the ACP/DCP in charge of the
drainage and embankments, water storage and water police station.The Authority will be required to dispose
power subject to the provisions of Entry 56 of List I’. an application seeking protection within five days from
The primary entry in the Constitution relating to water is the date of receipt of the Threat Analysis Report.
indeed Entry 17 in the State List, but it is explicitly made The Witness Protection Order passed by the Competent
subject to the provisions of Entry 56 in the Union List Authority shall be implemented by the Witness Protection
Cell of the state or Union Territory.
which enables the Union to deal with inter-State rivers if
It also envisaged the setting up of vulnerable witness
Parliament legislates for the purpose.
deposition complexes by the end of 2019. These rooms
Hence, statement 2 is NOT correct. will be equipped with facilities to prevent the accused
82. (c) and witness coming face to face.
Exp.: To ensure safety of witnesses, the Supreme Court recently 83. (c)
approved a Witness Protection Scheme. The question of Exp.: Evolution of TRAI and TDSAT:
witness protection had come up in a PIL that sought Telecom Regulatory Authority of India (TRAI) was
protection for witnesses in cases against Asaram Bapu. established in 1997. Apart from regulatory functions,
The court said that it shall be the ‘law’ under Article TRAI was also empowered to adjudicate upon disputes
141/142 of the Constitution, till the enactment of suitable among Service Providers or between the Service
Parliamentary and/or State Legislations on the subject. Providers and a group of Consumers.

641, First Floor, Dr. Mukherjee Nagar, Delhi-110009 19


Contacts% 011-47532596, (+91) 8448485517
E-mail: helpline@groupdrishti.com, Web: www.drishtiias.com Facebook: facebook.com/drishtithevisionfoundation
Copyright – Drishti The Vision Foundation
https://telegram.me/pdf4exams https://telegram.me/ias201819

The adjudicatory powers of TRAI were challenged before smaller but higher and the other much larger but lower.
the Delhi High Court on the ground that TRAI did not Archaeologists designate these as the Citadel and the
possess jurisdiction to issue directions to Department of Lower Town respectively. The Citadel owes its height to
Telecom (DoT) in the latter’s capacity as Licensor. The the fact that buildings were constructed on mud brick
Court held that TRAI does not possess the power to issue platforms. It was walled, which meant that it was
directions to the Government in latter’s capacity as physically separated from the Lower Town. The Lower
Licensor. Town was also walled. Hence , statement 3 is correct.
TRAI Act of 1997 was amended in the year 2000 and
85. (c)
Telecom Disputes Settlement and Appellate Tribunal
(TDSAT) was set up to adjudicate disputes and dispose Exp.: Endangered species such as the lion-tailed Macaque,
of appeals related to telecom sectors. Nilgiri Tahr and Nilgiri Langur are unique to the area
Later, Government included broadcasting, cable services of Western Ghats. Western Ghats is a hotspot of
and matters related to IT and Airport tariff within the Biodiversity in India. Hence, statement 1 is correct.
purview of TDSAT. Hence, statement 1 is correct. zzIt is also among the top eight biodiversity hotspots in the
The Information Technology Act, 2008 and the Airport world. The Western Ghats host over 400 species and seven
Economic Regulatory Authority of India Act, 2008. distinct vegetation types. Hence, statement 2 is correct.
The Tribunal exercises original as well as appellate
86. (d)
jurisdiction in regard to Telecom, Broadcasting and
Airport tariff matters. In regard to Cyber matters the Exp.: The biodiversity of the Ganga River is under stress due
Tribunal exercises only the appellate jurisdiction. to reduced water availability, habitat degradation,
In respect of Telecom, Broadcasting and Airport tariff pollution, and unsustainable resource extraction.
matters, the Tribunal’s orders can be appealed to the zzTo conserve the ecological integrity of the Ganga River,
Supreme Court but only on substantial questions of law. and, reduce the direct dependency of the local communities
Whereas in regard to Cyber matters, the Tribunal’s order on the river, the National Mission for Clean Ganga-
can be appealed before High Court. Wildlife Institute of India (NMCG-WII) project
Hence, statement 2 is correct. ‘Biodiversity Conservation and Ganga Rejuvenation’ is
84. (b) involving members of the local community in the five
Exp.: Grains found at Harappan sites include wheat, barley, Ganga states (Uttarakhand, Uttar Pradesh, Bihar,
lentil, chickpea and sesame. Millets are found from sites Jharkhand and West Bengal) as guardians of the river who
in Gujarat. Finds of rice are relatively rare. There are no will henceforth be known as Ganga Praharis. Hence,
evidences of cultivation of rice at large scale. Hence, statement 1 is NOT correct.
statement 1 is NOT correct. zzThe aim of this initiative is to establish a motivated cadre
zzMost Harappan sites are located in semi-arid lands, where of ‘Ganga Prahari’ to support the local level institutions
irrigation was probably required for agriculture. Traces and monitor the quality of the natural resources of the
of canals have been found at the Harappan site of river by mobilizing local communities at the grassroots
Shortughai in Afghanistan, but not in Punjab or Sind. It level. This can be achieved by:
is possible that ancient canals silted up long ago. It is also „„Creating awareness about the benefits of a clean and

likely that water drawn from wells was used for irrigation. vibrant Ganga and create a sense of belongingness
Besides, water reservoirs found in Dholavira (Gujarat) among people towards the Ganga River.
may have been used to store water for agriculture. Hence, zzLinking local communities and their livelihoods with the
statement 2 is correct. overall efforts of various agencies working for a clean
zzPerhaps the most unique feature of the Harappan Ganga, and thereby, creating a convergence point at grass
civilisation was the development of urban centres. The roots level for such efforts.
Mohenjodaro settlement is divided into two sections, one Hence, statement 2 is NOT correct.

641, First Floor, Dr. Mukherjee Nagar, Delhi-110009 20


Contacts% 011-47532596, (+91) 8448485517
E-mail: drishtiacademy@gmail.com, Web: www.drishtiias.com Facebook: facebook.com/drishtithevisionfoundation
Copyright – Drishti The Vision Foundation
https://telegram.me/pdf4exams https://telegram.me/ias201819

87. (c) 89. (c)


Exp.: On 5th June 1972, environment was first discussed as an Exp.: Comprehensive Environmental Pollution Index
item of international agenda in the U.N. Conference of To curb environmental pollution from Industrial areas
Human Environment in Stockholm and thereafter 5th Central Pollution Control Board (CPCB) developed
Comprehensive Environmental Pollution Index (CEPI)
June is celebrated all over the world as World Environment
to find out an Index value to characterise quality of the
Day. environment. Hence, statement 1 is correct.
zzIn India, the Wildlife (Protection) Act was passed in 1972, (CEPI) is a rational number to characterise the
followed by the Water (Prevention and Control of environmental quality at a given location following the
Pollution} Act 1974, the Forest (Conservation) Act, 1980, algorithm of, source, pathway, receptor and various
Air (Prevention and Control of Pollution) Act, 1981 and parameters like pollutant concentration, impact on human
health and level of exposure have been taken into
subsequently the Environment (Protection) Act,·1986.
consideration for the calculation of pollution indices for
Constitutional Provisions air, water and land.
Article-48-A of the constitution provides: The present CEPI is intended to act as an early warning
"The state shall endeavor to protect, and improve the tool. It can help in categorizing the industrial clusters. In
environment and to safeguard forest and wildlife of the terms of priority of planning needs for interventions.
country." Hence, statement 1 is correct. CPCB has revised CEPI concept in concurrence with
Article 51-A (g) MoEF&CC and subsequently issued directions on to all
Provides: “It shall be duty of every citizen off India to SPCBs/PCCs having CPAs for adoption of the revised
CEPI concept, which includes :
protect and improve the natural environment including
zzEnvironmental quality monitoring in all the CPAs,
forests, lakes, rivers and wildlife an to have compassion
zzInstallation of Continuous Ambient Air Quality
for living creatures.” Hence, statement 2 is correct.
Monitoring Stations,
88. (b) zzInstallation of Continuous Water Quality Monitoring
Exp.: The Wildlife (Protection) Act of 1972 Stations, Hence, statement 2 is correct.
The Act with its various amendments provides the 90. (b)
necessary tool to prevent damage to the wildlife. Exp.: Environment (Protection) Act,1986
The Wildlife (Protection) Act of 1972 provides the basic zzThe genesis of the Environmental {Protection) Act, 1986,
framework to ensure the protection and management of is in Article 48A {Directive Pclnciples of State Policy)
wildlife. The Act was amended subsequently in 1982, and Article 51A (g) (Fundamental Duties) of the Indian
1986, 1991 and 1993 to accommodate provision for its Constitution.
effective implementation. zzThe Act prescribes a special procedure for handling
zzThe rating of the Schedules I to V is in accordance with
hazardous substances and the concerned person has to
handle the hazardous substances according to the
the risk of survival of the wildlife {fauna} enlisted in
procedure of the Act.
them. The schedule VI has been added to include the
zzIn the commission of the offence under this Act by
specified, plant species to be protected by the Wildlife Government Department, the Act holds the Head of the
Protection Amendment Act of 1991. Department as guilty of the offence unless the head of
zzWith the amendment of the Act in 1991, powers of the the Department proves that the offence was committed
State Governments have been withdrawn almost totally, without his knowledge or that he exercised all due
Now the State Governments are not empowered to declare diligence to prevent the commission of such offence.
Hence, statement 1 is NOT correct.
any wild animal a vermin. Further by addition of
The Act consists of and deals with more stringent penal
provision, immunization of livestock within a radius of
provisions. The minimum penalty for contravention or
5 km from a National Park or sanctuary has been made violation of any provision of the law is an imprisonment
compulsory. Hence, statement 1 is NOT correct while for a term which may extend to five years or fine up to
statement 2 is correct. one lakh rupees, or both.

641, First Floor, Dr. Mukherjee Nagar, Delhi-110009 21


Contacts% 011-47532596, (+91) 8448485517
E-mail: helpline@groupdrishti.com, Web: www.drishtiias.com Facebook: facebook.com/drishtithevisionfoundation
Copyright – Drishti The Vision Foundation
https://telegram.me/pdf4exams https://telegram.me/ias201819

The Act also provides for the further penalty if the failure zzThis initiative has the potential to contribute towards the
or contravention continues after the date of conviction. realization of the Millennium Development Goals
It is `5000/- per day. If the failure of contravention (MDGs) by improving energy access for the rural poor.
continues beyond the period of one year, then the offender zzFormation of more than 100 women-led Self-Help Groups
is punished with imprisonment for a term which may (SHGs), and strengthening of around 150 SHGs are
extend to seven years. Hence, statement 2 is correct. among the impacts of this initiative.
91. (b) zzThe campaign has demonstrated how Public- Private-
People partnerships can support rural development
Exp.: The Biological Diversity Act 2002
schemes, particularly in the areas of health, education,
zzThe Biological Diversity Act 2002 was born out of India's
environment and women's empowerment.
attempt to realize the objectives enshrined in the United
zzThe campaign has drawn support from public sector units
Nations Convention on Biological Diversity (CBD) 1992
and corporates, among its various partners, to aid the
which recognizes the sovereign rights of states to use their
execution of the program at the scale at which it exists
own Biological Resources.
today.
zzIt provides for conservation of biological diversity,
sustainable use of its components and fair and equitable 93. (b)
sharing of the benefits arising out of the use of biological Exp.: National Green Tribunal (NGT)
resources, knowledge and for matters connected therewith The Preamble of the NGT Act, 2010 provides for the
or incidental thereto. establishment of a National Green Tribunal for the
Objectives effective and expeditious disposal of cases relating to
(i) Conservation of biological diversity; environmental protection and conservation of forests and
(ii) Sustainable use of its components; other natural resources.
(iii) Fair and equitable sharing of the benefits arising NGT includes cases relating to enforcement of any legal
from the utilization of genetic resources. Hence, right relating to environment and giving relief and
statement 2 is correct. compensation for damages to persons and property and
for matters connected there with or incidental there to
zzThe Act envisages a three-tier structure to regulate
(The National Green Tribunal Act, 2010). Hence,
access to the biological resources, comprising of National
statement 1 is NOT correct.
Biodiversity Authority (NBA), State Biodiversity Boards
With the establishment of the NGT, India has joined the
(SBB) and Biodiversity Management Committees (BMC)
distinguished league of countries that have a dedicated
at the local level. Hence, statement 1 is NOT correct.
adjudicatory forum to address environmental disputes.
92. (c) India is third country in the world to full fledged green
Exp.: LIGHTING A BILLION LIVES (LaBL). tribunal followed by new Zealand and Australia.
The specialized architecture of the NGT will facilitate
zzLaBL is a campaign by The Energy and Resources
fast track resolution of environmental cases and provide
Institute (TERI) that promotes the use of solar lanterns
a boost to the implementation of many sustainable
specially designed and manufactured on a decentralized
development measures.
basis. Hence, statement 1 is correct.
NGT is mandated to dispose the cases within six months
zzLaBL has been able to engage with government
of their respective appeals. Hence, statement 2 is
interventions under Sarva Shiksha Abhiyan, Madhya correct.
Pradesh Rural Livelihood Project, Rasthriya Gramin
Vikas Nidhi and has facilitated the spread of mobile 94. (b)
telephony with support from Department of Exp.: NATIONAL GANGA RIVER BASIN AUTHORITY
Telecommunications, Government of India. (NGRBA)
zzLaBL has successfully engaged the private sector and zzNGRBA was constituted on February 2009 under the
leveraged Corporate Social Responsibility (CSR). Hence, Environment (Protection) Act, 1986. Hence, statement
statement 2 is correct. 1 is NOT correct.
641, First Floor, Dr. Mukherjee Nagar, Delhi-110009 22
Contacts% 011-47532596, (+91) 8448485517
E-mail: drishtiacademy@gmail.com, Web: www.drishtiias.com Facebook: facebook.com/drishtithevisionfoundation
Copyright – Drishti The Vision Foundation
https://telegram.me/pdf4exams https://telegram.me/ias201819

zzNGRBA is mandated to take up regulatory and zzHorizontally Differentiated Banking System (HDBS)
developmental functions with sustainability needs for In a HDBS design, the basic design element remains a
effective abatement of pollution and conservation of the full-service bank that combines all three building blocks
river Ganga by adopting a river basin approach for of payments, deposits, and credit but is differentiated
comprehensive planning and management. primarily on the dimension of size or geography or
zzThe Ministry of Water Resources, River Development sectoral focus. Hence, statement 1 is correct.
and Ganga Rejuvenation (MoWR, RD & GR) is the nodal
zzVertically Differentiated Banking System (VDBS)
Ministry for the NGRBA.
In a VDBS design, the full-service bank is replaced by
zzThe authority is chaired by the Prime Minister and has as banks that specialise in one or more of the building blocks
its members the Union Ministers concerned, the Chief of payments, deposits, and credit. Hence, statement 2 is
Ministers of the States through which Ganga flows, viz.,
correct.
Uttarakhand, Uttar Pradesh, Bihar, Jharkhand and West
zzSome of the examples of such niche and specialised
Bengal, among others.
institutions are the South Korean Post Office Bank (only
zzThis initiative is expected to rejuvenate the collective
payments and deposits), GE Capital (credit and
efforts of the Centre and the States for cleaning the river.
payments), MasterCard and Visa (only payments).
zzIt is mandated to ensure the maintenance of minimum
ecological flows in the river Ganga and abate pollution 96. (c)
through planning, financing and execution of programmes Exp.: The Ministry of Road Transport & Highways has
including that of– recently launched the BhoomiRashi portal.
(1) Augmentation of Sewerage Infrastructure Hence, statement 2 is correct.
(2) Catchment Area Treatment zzIn the past, acquisition of land for the purpose of National
(3) Protection of Flood Plains Highway projects, payment of compensation to the land
(4) Creating Public Awareness owners, etc were done manually by physical movement
News:- The National Ganga Council headed by Prime of documents in the form of files.
Minister is tasked with the protection and upkeep of the zzHowever, in that procedure some constraints viz. delay
Ganga river, has convened no meetings since it was first
in issuing notification, errors in the land/area details, etc.
set up in 2016. However it was mandated to meet at least
were being faced.
once a year. Hence, statement 2 is correct.
Info- The Water (Prevention and Control of Pollution) zzIn order to overcome these issues, to cut short delays and
Act was enacted in 1974 to provide for the prevention avoid parking of public funds with the Competent
and control of water pollution, and for the maintaining Authority for Land Acquisition (CALA), Ministry has
or restoring of wholesomeness of water in the country. developed a web based Utility –BhoomiRashi to fully
digitize and automate the entire process of land
95. (c)
acquisition. Hence, statement 1 is correct.
Exp.: Differentiated banks are distinct from universal banks as
zzComprehensive details including names of 711 districts
they function in a niche segment. The differentiation could
and 6,55,297 villages have been built into the system
be on account of capital requirement, scope of activities
based on the population Census-2011, duly updated on
or area of operations. As such, they offer a limited range
the basis of revenue records of all the State Governments.
of services / products or function under a different
regulatory dispensation. The concept is not entirely new. zzThe system helps in expediting the process by providing
In fact, and in a sense, the UCBs, the PACS, the RRBs simultaneous Hindi translation and has been linked to the
and LABs could be considered as differentiated banks as e-gazette for expeditious publication.
they operate in localized areas. zzWith the operation of this Portal, the land acquisition
zzRBI has come up with two broad designs based on the process has been expedited significantly, become error-
functional building blocks of payments, deposits and free and more transparent and the notifications at every
credit for differentiated banks in the country stage are being processed on real time basis.

641, First Floor, Dr. Mukherjee Nagar, Delhi-110009 23


Contacts% 011-47532596, (+91) 8448485517
E-mail: helpline@groupdrishti.com, Web: www.drishtiias.com Facebook: facebook.com/drishtithevisionfoundation
Copyright – Drishti The Vision Foundation
https://telegram.me/pdf4exams https://telegram.me/ias201819

zzWhile the physical processing of the cases usually took 99. (b)
considerable time, ranging from weeks to even months, Exp.: A fugitive economic offender is any individual against
and there were errors whose rectification caused further whom warrants for arrest is issued for his involvement
delays, the processing time using this portal has been in select economic offences involving amount of at least
reduced to less than two weeks in a majority of cases and `100 crore or more and has left India so as to avoid
even a few days in some cases. criminal prosecution. Hence, statement 1 is NOT
correct.
97. (d)
The new law allows designated special court to declare
Exp.: The Inland Waterways Authority of India proposes to a person as fugitive economic offender and to confiscate
develop India's first “freight village” in Varanasi. The his property, including ‘benami’ ones. All the rights and
objective of the project is to support economic development title in the confiscated property shall, from the date of the
in the hinterland of the multimodal terminal at Varanasi confiscation order, vest in the Central government, free
and reduce logistics cost in the Eastern Transport Corridor from all encumbrances. Hence, statement 2 is correct.
and its influence zone. It envisages establishment of 100. (a)
multimodal logistics hub in Varanasi to promote the use
Exp.: The Reserve Bank of India’s (RBI) decision to allow
of waterway transport on river Ganga (NW-1) between card payment providers to offer tokenisation services will
Haldia and Varanasi and of rail transport on Eastern ensure the safety of digital transactions and reduce
Dedicated Freight Corridor (EDFC) in the North bound chances of fraud.
direction which facilitates movement of freight from road zzTokenisation involves a process in which a unique token
to water and rail. Hence, option (d) is correct. masks sensitive card details like card and CVV number.
98. (d) The token is used to perform card transactions in
contactless mode at Point Of Sale (POS) terminals, Quick
Exp.: Project “Sashakt” was proposed by a panel led by PNB
Response (QR) code payments, etc.
chairman Sunil Mehta. Bad loans of up to `50 crore will
zzThe way the token will work is like this — the debit or
be managed at the bank level, with a deadline of 90 days.
credit card holder will create a code for a particular
For bad loans of `50-500 crore, banks will enter an inter- amount, say `100, through an app in the form of a number.
creditor agreement, authorizing the lead bank to That number will have the amount that can be spent, the
implement a resolution plan in 180 days, or refer the asset merchant type where it can be spent as well as time within
to NCLT. For loans above `500 crore, the panel which the transaction needs to be completed. Then the
recommended an independent AMC, supported by number can be shared with the merchant who will enter
institutional funding through the AIF. The idea is to help it in the mobile, to get the payment for the items sold.
consolidate stressed assets. Hence, option (d) is correct. Hence, option (a) is correct.

641, First Floor, Dr. Mukherjee Nagar, Delhi-110009 24


Contacts% 011-47532596, (+91) 8448485517
E-mail: drishtiacademy@gmail.com, Web: www.drishtiias.com Facebook: facebook.com/drishtithevisionfoundation
Copyright – Drishti The Vision Foundation

You might also like